Need problem please please

Need Problem Please Please

Answers

Answer 1

The formula for the nth term of the geometric sequence is given as follows:

[tex]a_n = 7(-3)^{n - 1}[/tex]

What is a geometric sequence?

A geometric sequence is a sequence of numbers where each term is obtained by multiplying the previous term by a fixed number called the common ratio q.

The formula for the nth term of the sequence is given as follows:

[tex]a_n = a_1q^{n-1}[/tex]

The first term of the sequence is given as follows:

[tex]a_1 = 7[/tex]

Each term is the previous term multiplied by -3, hence the common ratio is given as follows:

q = -3.

Thus the formula for the sequence is given as follows:

[tex]a_n = 7(-3)^{n - 1}[/tex]

More can be learned about geometric sequences at https://brainly.com/question/24643676

#SPJ1


Related Questions

please tell methe cords of the answer

Answers

Answer:

(- 2, - 4 )

Step-by-step explanation:

4x - 3y = = 4 → (1)

3x + 3y = - 18 → (2)

adding the equations term by term will eliminate y

(4x + 3x) + (- 3y + 3y) = 4 - 18

7x + 0 = - 14

7x = - 14 ( divide both sides by 7 )

x = - 2

substitute x = - 2 into either of the 2 equations and solve for y

substituting into (2)

3(- 2) + 3y = - 18

- 6 + 3y = - 18 ( add 6 to both sides )

3y = - 12 ( divide both sides by 3 )

y = - 4

solution is (- 2, - 4 )

Classify the following into finite and infinite set
(i) The set of months in a year
(ii)The set of stars in the galaxy
(iii)The set of even integers
(iv)The set of odd integers
(v)The set of alphabets

Answers

i) finite set, ii) infinite set, iii) infinite set, iv) infinite set, and v) finite set are the required solutions.

A Set is a collection of objects with similar properties. There are different types of set two of them are finite and infinite set.

Finite set- A set is called a finite set when the elements inside the set are countable or where we can know the starting and the ending of the elements. For example- A set of players in a cricket team.

Infinite set- A set is called an infinite set when the elements inside the set are uncountable or where the ending of the elements is not known. For Example- A set of positive numbers.

Here in the question the option (i) and (v) are finite sets because the no.of months in the year and no.of alphabets are countable or finite. Option (ii), (iii), and (iv) are infinite sets as no.of stars in the galaxy, even integers, and odd integers are uncountable.

Learn more about Sets at:

https://brainly.com/question/28574826

Answer:

the set of months in a year is finite

the set of star in the Galaxy is finite

the set of even integers is infinite

the set of odd integers is infinite

the set of alphabet is finite

Step-by-step explanation:

1) the set of month in a year is finite because they are limit and we can count them easily

2) the set of star in the galaxy is finite cause there are zillions of stars

3) the set of even integer is infinite because numbers are unlimit or infinite

4) the set of odd integers is infinite cause numbers are infinite ir they have not limit

5) the set of alphabet is finite because we can count them limitedly

How much will you have in our retirement account at the end of 2 years if you contribute $3,000 a year and have a 100% employer match? Assume a 9% annual return on your investments and end-of-year contributions.

0 $12,540.00
0 $6,000.00

Answers

Assuming that the $3,000 annual contribution is made at the end of each year, and that the 100% employer match is also made at the end of each year, the total contribution for each year would be $6,000 ($3,000 from you and $3,000 from your employer).

At a 9% annual return, the future value of each $6,000 contribution at the end of two years would be:

Year 1: FV = $6,000 x 1.09 = $6,540
Year 2: FV = ($6,000 + $6,540) x 1.09 = $13,080

Therefore, the total amount in your retirement account at the end of two years would be $13,080.

Can u mark my answer as the Brainlyest if it work Ty

8 cupcakes in 1 box = 16 cupcake in X boxes

Answers

Answer:

2 boxes

Step-by-step explanation:

8 cupcakes in 1 box = 16 cupcake in x box

We Take

16 ÷ 8 = 2 boxes

So, the answer is 2 boxes.

Answer: 2 boxes
Explanation: There are 8 cupcakes in 1 box. If there are 16 cupcakes, (8x2=16) there are 2 boxes.

Hurry up please (time limit)

State the domain and range and tell if the graph is a function yes or no?
What’s the domain and range?

Answers

The graph in this problem is not a function, as it contains vertically aligned points.

The domain and the range are given as follows:

Domain: -3 ≤ x ≤ 3.Range: 1 ≤ y ≤ 3.

When does a graph represents a function?

A graph represents a function if it has no vertically aligned points, that is, each value of x is mapped to only one value of y. Vertically aligned points mean that a value of x is mapped to multiple values of y, that is, a single input is mapped to multiple outputs which disqualify the relation as a function.

When we trace a vertical line through the circle, it would cross the circle at two points, meaning that the graph contains vertically aligned points, and thus, is not a function.

The domain and the range are defined as follows:

Domain: -3 ≤ x ≤ 3. -> values of x on the graph.Range: 1 ≤ y ≤ 3. -> values of y on the graph.

More can be learned about graphs and functions at https://brainly.com/question/12463448

#SPJ1

QUESTION ONE
(a) If the mean of five values is64, find the sum of the values
(b) If the mean of five values is 8.2 and four of the values are 6, 10, 7, and 12, find the fifth vaue.
(c) Find the mean of 10, 20, 30, 40, and 50.
i) Add 10 to each value and find the mean.
ii) Subtract 10 from each value and find the mean.
iii) Multiply each value by 10 and find the mean.
iv) Divideeach value by 10 and find the mean.
v) Make a general statement about each situation.

QUESTION TWO
a)Find the mean deviation for these data. 5, 9, 10, 11, 11, 12, 15, 18, 20, 22

b)The mean marks in statistics of 100 students in a class was 72 per cent. The mean marks of boys
was 75 per cent, while their number was 70 per cent. Find outthe mean marks of girls in the
cass.

c)The weighted geometric mean of the four numbers 20, 18, 12, and 4 is 11.75. If the weights of
the first three numbers are 1, 3, and 4, respectivey, find the weight of the fourth number

QUESTION THREE
a)For a group of 50 male workers, the mean and standard deviation of their monthly wages
are $6300 and $900 respectively. For a group of 40 female workers, these are $ 5400 and $
600 respectively. Find the standard deviation of monthly wages for the combined group of
workers.

b)A study of the age of 100 persons grouped into intervals 20–22, 22–24, 24–26..., revealed
the mean age and standard deviation to be 32.02 and 13.18 respectivey. While checking it
was discovered that the observation 57 was misread as 27. Cacuate the correct mean age and standard deviation

QUESTION FOUR
The weekly sales of two products A and B were recorded as given below:
Product A : 59 75 27 63 27 28 56
Product B : 150 200 125 310 330 250 225
Find out which of the two shows greater fluctuation in sales

Answers

(a) Sum of values = Mean × Number of values = 64 × 5 = 320

(b) Sum of values = Mean × Number of values = 8.2 × 5 = 41

Fifth value = Sum of values - sum of known values = 41 - (6 + 10 + 7 + 12) = 41 - 35 = 6

How to solve

(c) The resulting middle number when computing 10, 20, 30, 40, 50;that is (10 + 20 + 30 + 40 + 50)/5, totals up to 150 and dividing that number by 5 reflects a mean of 30.

i) Forming an average of 20, 30, 40, 50, 60;that is (20 + 30 + 40 + 50 + 60)/5 provides an outcome of 200 and divided by 5 reveals a mean of 40.

ii) With 0, 10, 20, 30, 40 used, (0 + 10 + 20 + 30 + 40)/5 practices calculation yielding a total of 100 which, split into five parties, imparts a mean of 20.

iii) Taking into consideration the input of 100, 200, 300, 400, 500;whereby (100 + 200 + 300 + 400 + 500)/5 develops a collective of 1500, cut in half produces a mean of 300.

iv) Calculating 1, 2, 3, 4, 5's mean; namely (1 + 2 + 3 + 4 + 5)/5 ascertains a collective amount of 15, divide that figure by five culminates with a mean of 3.

v) Accordingly, the following general assumptions can be made:

i) Introducing a constant to each value augments the mean by that very same constant.

ii) Extracting a constant from all values decrements the mean by such said constant.

iii) Multiplying every value by one constant multiplies the mean by the same specific constant.

iv) Dividing all values by a particular constant divides the mean by that given constant.

Read more about mean here:

https://brainly.com/question/1136789

#SPJ1

3 dependent events. A bag contains ten balls. Six of the balls are green. A ball is chosen at random. The ball is replaced and then a second ball is chosen at random. Work out the probability that: a)both balls are green b)neither ball is green​

Answers

The Probability of both balls being green is  0.36 or 36%

b) The Probability of neither ball being green is 0.16 or 16%

What is the Probability  about?

Based on question A,  To find the probability that both balls are green, we use the equation below:

P (both green) = P (first green) x P (second green)

= (6/10) x (6/10)

= 36/100

= 9/25

=0.36

b) To find the probability that neither ball is green,  we use the formula below:

P (neither green) = P (first not green) x P (second not green)

= (4/10) x (4/10)

= 16/100

= 4/25

=0.16.

Therefore, the statement will be:  The Probability of both balls being green: 0.36 and the probability of neither ball being green is 0.16

Learn more about Probability from

https://brainly.com/question/24756209

#SPJ1

Volume of Prism 2 3/4, 5 1/2, 3 2/5

Answers

Volume of the prism given is 51 [tex]\frac{17}{40}[/tex].

The volume of the prism is given by the formula,

Volume of the prism = Base area × height

Base area = 2 3/4 × 5 1/2

                 = 11/4 × 11/2

                 = 121 / 8

Volume of the prism = 121 / 8 × 3 2/5

                                  = 121/8 × 17/5

                                  = 2057/40

                                  = 51 17/40

Hence the volume of the prism is 51 [tex]\frac{17}{40}[/tex].

Learn more about Volume here :

https://brainly.com/question/13338592

#SPJ1

The volume of a cone is 109.95 cubic inches and the height is 4.2 inches. What is the radius of the cone?

Answers

The calculated radius of the cone is 5 inches

What is the radius of the cone?

From the question, we have the following parameters that can be used in our computation:

Volume = 109.95 cubic inches

Height = 4.2 inches.

The volume is calculated as

V = 1/3 * 3.14 * r^2h

So, we have

1/3 * 3.14 * r^2 * 4.2 = 109.95

So, we have

r^2 = 3 * 109.95/(4.2 * 3.14)

So, we have

r^2 = 25.01

Take the square roots

r = 5

Hence, the radius is 5 inches

Read more about volume at

https://brainly.com/question/463363

#SPJ1

Is anyone good with Math or History

Answers

The probability of passing at least one will be 0.70.

We know that,

Its basic premise is that something will almost certainly happen. The percentage of favorable events to the total number of occurrences.

here, we have,

Probability of passing history course = 0.59

Probability of passing math course = 0.60

Probability of passing both courses = 0.49

so, we get,

Probability of passing at least one will be

⇒ 0.59 + 0.60 – 0.49

⇒ 0.70

More about the probability link is given below.

brainly.com/question/795909

#SPJ1

complete question:

A student is taking two courses, history and math. the probability the student will pass the history course is 0.59, and the probability of passing the math course is 0.60. the probability of passing both is 0.49. what is the probability of passing at least one?

What is the area of the reduced rectangle? 0.18 square units 1.8 square units 18 square units 180 square units

Answers

The  area of the reduced rectangle is 1.8 unit²

The correct answer is an option (b)

Here the length of the rectangle is 15 units.

And the width of the rectangle is 12 units.

The rectangle has been reduced by a scale of 1/10

so, the new length of the rectangle is:

l = 1/10 × 15

  = 1.5 units

And the new width of the rectangle is:

w = 1/10 × 12

   = 1.2 units

We need to determine the area of the reduced rectangle.

Using the formula for area of rectagle, the area of the reduced rectangle would be:

A = l × w

A = 1.5 × 1.2

A = 1.8 unit²

Therefore, the correct answer is an option (b)

Learn more about the area of triangle here:

https://brainly.com/question/19305981

#SPJ1

The complete question is:

The rectangle below has been reduced by a scale of StartFraction 1 over 10 EndFraction. A rectangle has a length of 15 and width of 12. [Not drawn to scale] What is the area of the reduced rectangle?

a) 0.18 square units

b) 1.8 square units

c) 18 square units

d) 180 square units

The function f(x) = 1.85x^2 models the cost of a square carpet, where x is the length in feet. Find the average rate of change for f, to the nearest tenth, over the interval 10 ≤ x ≤ 20.

Answers

Answer:

I helped you last time so I'll help you again.

Step-by-step explanation:

We can find the average rate of change of f(x) over the interval [10, 20] using the formula:

average rate of change = (f(b) - f(a))/(b - a)

where a = 10 and b = 20.

So, we have:

f(a) = f(10) = 1.85(10)^2 = 185

f(b) = f(20) = 1.85(20)^2 = 740

average rate of change = (740 - 185)/(20 - 10) ≈ 55.5

Therefore, the average rate of change of f(x) over the interval [10, 20] is approximately 55.5.

Look at the picture

Answers

According to the information, we can infer that the area of the triangle is: 19 1/4ft²

How to calculate the area of the triangle?

To calculate the area of the triangle, we need to use the formula for the area of a triangle:

Area = (1/2) x base x height

where,

Base = 5 1/2 ftHeight = 7 ft.

How  to find the height of the triangle?

To find the height of the triangle, we need to use the Pythagorean theorem, which relates the slant height, height, and half the base:

(slant height)^2 = (height)^2 + (1/2 base)^2

Substituting the given values, we get:

(1/3)^2 = (7)^2 + (1/2 x 5 1/2)^2(1/9) = 49 + (15/4)(1/9) = (196/4 + 15/4)(1/9) = (211/4)

Multiplying both sides by 36, we get:

4 = 844.44...

So the height of the triangle is approximately 2.04 ft.

Now we can plug in the values for the base and height into the formula for the area:

Area = (1/2) x 5 1/2 x 7Area = 19.25 square feetArea = 19 1/4ft²

Therefore, the area of the triangle is approximately 19.25 square feet.

Learn more about area in: https://brainly.com/question/27683633

#SPJ1

A show box measures 15 in. by 7 in. by 4 1/2 in. What is the surface of the box?

Answers

The surface area of the box is 408 square inches whose dimensions of the box are 15 in. by 7 in. by [tex]4\frac{1}{2}[/tex] in.

The formula for the surface area of a rectangular box is:

Surface Area = 2lw + 2lh + 2wh

where l, w, and h are the length, width, and height of the box.

Given that the dimensions of the box are 15 in. by 7 in. by [tex]4\frac{1}{2}[/tex] in., we have:

l = 15 in.

w = 7 in.

h = [tex]4\frac{1}{2}[/tex] in. = 9/2 in.

Substituting these values into the formula, we get:

Surface Area = 2lw + 2lh + 2wh

Surface Area = 2(15)(7) + 2(15)(9/2) + 2(7)(9/2)

Surface Area = 210 + 135 + 63

Surface Area = 408

Therefore, the surface area of the box is 408 square inches.

To learn more on Three dimensional figure click:

https://brainly.com/question/2400003

#SPJ1

A sack of rice wieghs 50 kilos. What is the weight of 85 sacks of rice?

Answers

Answer:

4250 kilos

Step-by-step explanation:

Match these values of r with the accompanying​ scatterplots:

−0.405​,

−0.998​,

−1​,

−0.743​,

and

0.743.

Answers

The matching table is given in the image below:

What is a Scatter Plot?

A scatter plot, an undeniably essential chart for visualizing the correlation between two variables, is constructed by putting one variable on the x-axis and another one alongside the y-axis, with each individual data point symbolizing both associated values.

Scatter plots are not only indispensable when inspecting trends and peculiarities in a given set of data but can likewise be applied to examine the relationship or link between the two components.

Essentially imperative to fields such as economics, engineering, and statistics, these diagrams are efficiently drawn up through popular software applications like Excel or Python.

Read more about scatter plots here:

https://brainly.com/question/6592115

#SPJ1

Please help me solve questions 14,15, & 16.

Answers

Answers in bold:14.  probability = 2/13  (choice D)15.  probability = 4/663 (choice C)16.  probability = 3/52 (choice B)

=================================================

Explanation for problem 14

There are 4 aces and 4 copies of "9".

4+4 = 8 cards we want to select out of 52 total

8/52 = (4*2)/(4*13) = 2/13  which points to choice D as the final answer

---------------

Explanation for problem 15

4 kings gives a probability of 4/52 = 1/13.

After the king is chosen, there are 52-1 = 51 cards left since we aren't replacing it. The probability of getting a queen on the second selection is 4/51

(1/13)*(4/51) = 4/663 is the probability of a king, then a queen, with no replacement. We arrive at choice C as the final answer.

----------------

Explanation for problem 16

face card = picture card

face card = Jack, Queen, King

aces are not considered a face card

3*4 = 12 face cards in total

A = 12/52 = (4*3)/(4*13) = 3/13 is the probability of selecting a face card.

That card is then put back (aka replacement) and we still have 52 cards in the deck. We don't drop down to 51.

13 hearts ---> B = 13/52 = 1/4 is the probability of getting a heart

A*B = (3/13)*(1/4) = 3/52 is the probability of getting a face card, then a heart, when replacement is done. The final answer is choice B

A hippopotamus weighs 2 tons and a crocodile weighs 20 lbs.

How many times as much as the hippopotamus does the crocodile weigh?

Answers

The crocodile weighs 0.005 times as much as the hippopotamus.

To determine how many times as much as the hippopotamus a crocodile weighs, we need to compare the weights of both animals. However, they are currently in different units, so we need to convert them to a common unit.

We can convert the weight of the hippopotamus from tons to pounds by multiplying by 2,000, since there are 2,000 pounds in a ton. This gives us:

Hippopotamus weight = 2 tons × 2,000 pounds/ton = 4,000 pounds

Now that both weights are in pounds, we can compare them. To determine how many times as much as the hippopotamus the crocodile weighs, we can divide the weight of the crocodile by the weight of the hippopotamus. This gives us:

Crocodile weight / Hippopotamus weight = 20 pounds / 4,000 pounds = 0.005

Alternatively, we could say that the hippopotamus weighs 200 times as much as the crocodile.

To learn more about weight click on,

https://brainly.com/question/9182431

#SPJ1

picture look in the picture

Answers

Answer:

The measure of angle E is 56°.

What is the sum of 3.0 x 4.1 x 5.5

Answers

Answer: 67.65

Step-by-step explanation:

Multiply 3.0*4.1 and you get 12.3

Then take that sum and multiply it by the 5.5

So 12.3*5.5=67.65

The sum of 3.0 x 4.1 x 5.5 is equal to 67.65.

Ms. Monti bought * adult tickets and y
children's tickets to an ice-skating show. She spent a total of $145. The equation below describes the relationship
between x and y.
25x + 15y = 145 The ordered pair (4, 3) is a solution of the equation. What does the solution
(4, 3) represent?
A Ms. Monti bought 4 adult tickets and
3 children's tickets. B Adult tickets cost $4 each and
children's tickets cost $3 each. C Ms. Monti spent $4 on adult tickets
and $3 on children's tickets. D The cost of 4 adult tickets equals the
cost of 3 children's tickets.

Answers

A Ms. Monti bought 4 adult tickets and 3 children's tickets.

The ordered pair (4, 3) represents the number of adult tickets and children's tickets that Ms. Monti bought. Since the equation 25x + 15y = 145 is true when x = 4 and y = 3, this means that Ms. Monti bought 4 adult tickets and 3 children's tickets, and spent a total of $145.

Vocabulary
1. The remainder is the number that remains after the division is
complete. Use an R to indicate the remainder.
Tia has 26 walnuts. She gives 7 walnuts to each friend. How many friends get
7 walnuts? How many walnuts are left over?
Use the array to find 26+7. Circle the groups of 7.
There are
groups of 7.
left over.
There are
2647-
friends each get 7 walnuts.
walnuts left over.
000000
000000
0.0
000
O
There are
2. Juan puts 57 oranges in bags. Each bag holds 6 oranges. Use the array to divide.
To find 57 +6, circle groups of
5746-
How many full bags of oranges are there?
How many oranges are not in bags?
How many bags does Juan need to put all the oranges in bags?.
3. How many craft sticks will be left over if 9 friends equally share a
package of 85 craft sticks?
000000
OOO
000
000
O
000000
000000
000000
000000
4. A group of 43 people are going to a concert. If 6 people fit in each
car, how many cars will they need to take?
000000
000000
On the Back!
6. Find the number of equal groups and the number left over for
88 +3. Show your work.
bags
000
5. Bess has 19 sunflowers that she is putting into vases. She will put 4 sunflowers in
each vase. How many vases will have 4 flowers?

Answers

The number of friends that get 7 walnuts is 3 and there are 5 walnuts are left over

Sharing walnuts among friends

Here, we have

Walnuts = 26Walnuts to each friend = 7

So, we have

Friends = 26/7 = 3 R 5

This means that 3 friends get 7 walnuts and there are 5 walnuts are left over

Putting oranges in bags

Here, we have

Oranges = 57

Oranges in each bags = 6

So, we have

Bags  57/6 = 9 R 3

This means that there are 9 full bags of oranges, 3 oranges are not in the bag and Juan needs 10 bags in total

Crafts in sticks

Here, we have

Craft sticks = 85

Friends = 9

So, we have

Sticks left = 85\9

Evaluate

Sticks left = 4

Hence, there will be 4 sticks left

Read more about quotients at

https://brainly.com/question/1807180

#SPJ1

Enter the correct answer in the box.
What is the factored form of this expression?
2x^3+5x^2+6x+15
Write all factors in standard form.

Answers

Answer:

(2x + 5)(x² + 3)

------------------

Given expression:

2x³ + 5x² + 6x + 15

Group the expression as below and factorize:

2x³ + 5x² + 6x + 15 = (2x³ + 5x²) + (6x + 15) =x²(2x + 5) + 3(2x + 5) = (2x + 5)(x² + 3)

Hence the factored form of the expression is (2x + 5)(x² + 3).

The factored form of the expression is,

⇒ (2x + 5)(x² + 3).

Given that;'

The expression is,

⇒ 2x³ + 5x² + 6x + 15

Now, Group the expression as below and factorize:

2x³ + 5x² + 6x + 15

(2x³ + 5x²) + (6x + 15)

x²(2x + 5) + 3(2x + 5)

(2x + 5)(x² + 3)

Hence, the factored form of the expression is (2x + 5)(x² + 3).

Learn more about the mathematical expression visit:

brainly.com/question/1859113

#SPJ1

HELP ME ASAP PLS. If 19,432 divided by x equals 48 with a remainder of 232. What is X?

Answers

The value of x, given that when it divides a number, there is a quotient and a remainder, is 400

How to find the value of x?

To find the value of divisor, we can use a variant of the division algorithm formula :

Divisor = ( Dividend - Remainder ) / Quotient

x is the divisor

Dividend = 19, 432

Remainder = 232

Quotient = 48

This means that the value of x is:

x = ( 19, 432 - 232 ) / 48

x = 19, 200 / 48

x = 400

In conclusion, x is 400.

Find out more on remainder at https://brainly.com/question/19336517

#SPJ1

(ILL GIVE YOU 100 POINTS!!)
The work of a student to solve the equation 4(2x − 4) = 8 + 2x + 8 is shown below: Step 1: 4(2x − 4) = 8 + 2x + 8
Step 2: 6x − 8 = 16 + 2x
Step 3: 6x − 2x = 16 + 8
Step 4: 4x = 24
Step 5: x = 6
In which step did the student first make an error and what is the correct step?

a Step 2; 8x − 4 = 2(6 + x + 6)

b Step 2; 8x − 16 = 16 + 2x

c Step 3; 6x − 2x = 16 − 8

d Step 3; 6x + 2x = 16 + 8

Answers

Answer:

step 2 is wrong

Step-by-step explanation:

4 multiply by 2x = 8x4 multiply by -4 = -16CORRECT STEP4(2x - 4) = 8 + 2x + 88x - 16 = 8 + 2x + 88x - 16 = 2x + 16 6x = 32x = 32/6 = 5 ( approximately )

Here, The work of a student to solve the equation :

[tex]4(2x - 4) = 8 + 2x + 8[/tex]

The steps for calculating the value of x are :

Step 1: [tex]4(2x - 4) = 8 + 2x + 8[/tex]

Step 2: [tex]6x - 8 = 16 + 2x[/tex]

Step 3: [tex]6x - 2x = 16 + 8[/tex]

Step 4: [tex]4x = 24[/tex]

Step 5: [tex]x = 6[/tex]

Step 2 is incorrect, as the student instead of multiplying, adds the number as:

[tex]4(2x - 4) = 8 + 2x + 8[/tex]

Here, the correct step is:

[tex]\boxed{\bold{8x - 16 = 16 + 2x}}[/tex]

[tex]6 x = 32[/tex]

[tex]x = 5.33[/tex]

So, the error is in STEP 2 and the correct option is (B).

What is the meaning of "the equation ax = b has a solution for every b ∈ H"?

Answers

In the given image, H is a non-empty set of complex numbers, a and b are complex numbers, and x is a variable.

"The equation ax = b has a solution for every b ∈ H" means that for any complex number b in the set H, there exists a complex number x such that the product of a and x is equal to b.

In other words, for every numbers of b in H, there exists a solution for x that satisfies the equation ax = b.

This statement has important implications in linear algebra and functional analysis, where it is used to define the concept of a surjective linear transformation in numbers. It also has applications in many areas of mathematics and science, including engineering, physics, and computer science.

For more details regarding equation, visit:

https://brainly.com/question/29657983

#SPJ1

Find the tangent of the smaller acute angle in a right triangle with side lengths 8, 15, and 17

Answers

Answer: the tangent of the smaller acute angle in this right triangle is 8/15.

Step-by-step explanation:

Since the side lengths of the correct triangle are given as 8, 15, and 17, able to see that the sides with lengths 8 and 15 are the legs of the proper triangle, and the side with length 17 is the hypotenuse.

We know that the littler intense point θ is inverse the shorter leg of length 8. Hence, the digression of θ is:

tan(θ) = opposite/adjacent = 8/15

Dingaka receives R12650R12650 when taking out a loan with an annual simple discount rate of 12,5%12,5%. The loan has to be repaid in eight months’ time. The amount that he has to repay is?​

Answers

Dingaka took out a loan of R12650 with a simple annual discount rate of 12.5%. He has to repay R11595.83 in eight months, after deducting the simple discount of R1054.17.

To find the amount that Dingaka has to repay, we can use the formula for simple discount

Simple Discount = Principal * Rate * Time

Where Principal is the original amount borrowed, Rate is the annual simple discount rate, and Time is the time period in years.

We know that Dingaka received R12650 as the principal amount and the annual simple discount rate is 12.5%, which is equivalent to 0.125 as a decimal. The time period is 8/12 = 2/3 years, since the loan has to be repaid in eight months' time.

Plugging these values into the formula, we get

Simple Discount = 12650 * 0.125 * 2/3

= 12650 * 0.08333333

= 1054.166625

Therefore, the amount that Dingaka has to repay is the principal minus the simple discount, which is

Amount to Repay = Principal - Simple Discount

= 12650 - 1054.166625

= 11595.833375

So, Dingaka has to repay R11595.83 after eight months.

To know more about loan:

https://brainly.com/question/30581687

#SPJ1

Using the picture, give the term that best describes
each of the following.
C.
1.
E
D
B
G
F
H
BC
3. DH
4. EH 5. OG 6. GF
7. B
2. Term not applicable to this diagram

Answers

The labelled parts are described as follows

1. BC - Tangent

3. DH - Chord

4. EH - Diameter

5. G - center of the circle

6. GF Radius of the circle

7. B. a point of tangency

What is tangent and chord of a circle

Tangent: Referenced as a point of tangency, a tangent to a circle is a line that meets the circumference at one solitary area while never penetrating inside its bounds. To explain further, this line is perpendicular to the radius of the circle right at the tip of contact.

Chord: Referring to such an element, a chord of a circle is a straight piece between two areas that lay along the circumference. Without having any relation to the middle of the target object, it has both of its extremities on the peripheral border.

Learn more about tangent and chord at

https://brainly.com/question/4470346

#SPJ1

b) Find the value of w. Sive each answer as an integer or as a fraction in its simplest form 4 cm A 7 cm 12 cm 3 cm B W cm 9 cm​

Answers

The calculated value of w from the similar triangles is 21/4

Calculating the value of w

From the question, we have the following parameters that can be used in our computation:

Similar triangles A and B

Also from the similar triangles, we have the following proportional relation

4    7     12

3    W     9

Using the above as a guide, we have the following equation

4/3  =  7/W  =   12/9

So, we have

W * 12 = 7 * 9

Evaluate the products and divide both sides by 12

W = 21/4

Hence, the value of W is 21/4

Read more about similar shapes at

https://brainly.com/question/14285697

#SPJ1

Other Questions
What characteristic of life? A cat is composed of organ systems, which are composed of organs, which are composed of tissues, which are composed of cells.a) sensitivityb) homeostasisc) ordered complexityd) reproductione) energy utilizationf) evolution Which of the following describe American ideological ideas, and which reflect the realities of American society?Ideals:-political and legal equality for all-political authority from the consent of the governed-religious toleration and freedom of thoughtRealities:-slavery-Indians ceded more land to white settlers-married women had virtually no rights at all 64. What happens when Beowulf strikes his enemy with his sword? People's attitudes about appliances such as vacuum cleaners and refrigerators are most likely ___________________ based. a) Cognitively b) Affectively c) Behavioural d) Dissonance What is the absolute value of 34 If an Agile team planned to complete 30 story points in an iteration but worked at only 83% of the rate planned. What is the project's SPI? why can the fashion industry potentially be viewed as an unreliable source of employment help pls anyone mark brainleist What is the order of events in The Tragedy of Macbeth? Label the events in the order in which theyoccur in the play. The first event will be labeled 1, and the last event will be labeled 5Macbeth sees the ghost at the table.Macbeth envisions the bloody dagger.Macbeth arranges the murder of Banquo.Witches prophesy that Macbeth will be king.Macbeth fights fiercely and improves his reputation. 1. [0. 6/2 Points] DETAILS PREVIOUS ANSWERS MY NOTES ASK YOUR TEACHER Problem 6-23 Consider a random experiment involving three boxes, each containing a mixture of red and green balls, with the following quantities: Box A Box B Box C 31 Red Balls 12 Red Balls 24 Red Balls 16 Green Balls 20 Green Balls 16 Green Balls The first ball will be selected at random from box A. If that ball is red, the second ball will be drawn from box B; otherwise, the second ball will be taken from box C. Let R1 and G1 represent the color of the first ball, R2 and G2 the color of the second. Determine the following probabilities. (Hint: The conditional probability identity will not work. ) (a) Pr[Ru]= 65957 (b) Pr[G]= 340425 (c) Pr[R2 | Ri]=. 247338 X (d) Pr[R2 | Gi]= (e) Pr[G2 | Gi]= (f) Pr[G2 | R]= A nurse is caring for a trauma victim who has experienced a significant blood loss. Immediately following multiple transfusions, what is the most accurate indicator of oxygenation?Pulse oximetryHemoglobin and hematocrit (H and H)Complete blood count (CBC)Arterial blood gases (ABGs) What happened after the Chinese Exclusion Act was passed? Find the area of the triangle. From the sample statistics, find the value of -, the point estimate of the difference of proportions. Unless otherwise indicated, round to the nearest thousandth when necessary. n1 = 100 n2 = 100 = 0.12 = 0.1 A. 0.22 B. none of these C. 0.02 D. 0.012 E. 0.002 A radioactive element has decayed to 1/32 of its original concentration in 10 yrs. What is the half-life of this element? What should Mr. Karteris be told if one of these interacting meds such as a benzodiazepine for anxiety must be prescribed for him? You decided that price will be the determining factor in choosing a new ERP system. You select the option with the lowest price. The executive team likes the price tag, but soon you realize that the cost was so low because every phase of the project requires additional charges for customer support As your team begins working on the migration to the new module, you discover a compatibility problem between the new POS system and the old inventory management system that you were planning on replacing later. After a great deal of research and with limited support from your new vendor, you realize you have two options: replace the inventory management system at the same time, or invest some money into adapting the old system so it will work until you can replace it later Consider the Bohr model of the atom. Which transition would correspond to the largest wavelength of light absorbed? Select one: O n=2 to n=6 n=6 to n=10 O n=1 to n=5 O n=6 to n=3 O n=4 to n=1 Which information supports the appropriateness of a nursing diagnosis?A. Defining characteristicsB. Planning interventionsC. Diagnostic statementD. Related risk factors What is the sum of 8 of the interior angles of a regular nonagon?